Difference between revisions of "2020 AMC 8 Problems/Problem 4"

m (Solution 4)
m (Solution 3 (Pattern of the Bands))
(25 intermediate revisions by 3 users not shown)
Line 3: Line 3:
  
 
<asy>
 
<asy>
 +
// diagram by SirCalcsALot, edited by MRENTHUSIASM
 
size(250);
 
size(250);
real side1 = 1.5;
+
path p = scale(0.8)*unitcircle;
real side2 = 4.0;
+
pair[] A;
real side3 = 6.5;
 
real pos = 2.5;
 
pair s1 = (-10,-2.19);
 
pair s2 = (15,2.19);
 
 
pen grey1 = rgb(100/256, 100/256, 100/256);
 
pen grey1 = rgb(100/256, 100/256, 100/256);
 
pen grey2 = rgb(183/256, 183/256, 183/256);
 
pen grey2 = rgb(183/256, 183/256, 183/256);
fill(circle(origin + s1, 1), grey1);
+
for (int i=0; i<7; ++i) { A[i] = rotate(60*i)*(1,0);}
for (int i = 0; i < 6; ++i) {
+
path hex = A[0]--A[1]--A[2]--A[3]--A[4]--A[5]--cycle;
draw(side1*dir(60*i)+s1--side1*dir(60*i-60)+s1,linewidth(1.25));
+
fill(p,grey1);
}
+
draw(scale(1.25)*hex,black+linewidth(1.25));
fill(circle(origin, 1), grey1);
+
pair S = 6A[0]+2A[1];
for (int i = 0; i < 6; ++i) {
+
fill(shift(S)*p,grey1);
fill(circle(pos*dir(60*i),1), grey2);
+
for (int i=0; i<6; ++i) { fill(shift(S+2*A[i])*p,grey2);}
draw(side2*dir(60*i)--side2*dir(60*i-60),linewidth(1.25));
+
draw(shift(S)*scale(3.25)*hex,black+linewidth(1.25));
}
+
pair T = 16A[0]+4A[1];
fill(circle(origin+s2, 1), grey1);
+
fill(shift(T)*p,grey1);
for (int i = 0; i < 6; ++i) {
+
for (int i=0; i<6; ++i) {  
fill(circle(pos*dir(60*i)+s2,1), grey2);
+
fill(shift(T+2*A[i])*p,grey2);
fill(circle(2*pos*dir(60*i)+s2,1), grey1);
+
fill(shift(T+4*A[i])*p,grey1);
fill(circle(sqrt(3)*pos*dir(60*i+30)+s2,1), grey1);
+
fill(shift(T+2*A[i]+2*A[i+1])*p,grey1);
draw(side3*dir(60*i)+s2--side3*dir(60*i-60)+s2,linewidth(1.25));
 
 
}
 
}
 +
draw(shift(T)*scale(5.25)*hex,black+linewidth(1.25));
 
</asy>
 
</asy>
  
 
<math>\textbf{(A) }35 \qquad \textbf{(B) }37 \qquad \textbf{(C) }39 \qquad \textbf{(D) }43 \qquad \textbf{(E) }49</math>
 
<math>\textbf{(A) }35 \qquad \textbf{(B) }37 \qquad \textbf{(C) }39 \qquad \textbf{(D) }43 \qquad \textbf{(E) }49</math>
  
==Solution 1==
+
==Solution 1 (Pattern of the Rows)==
  
 
Looking at the rows of each hexagon, we see that the first hexagon has <math>1</math> dot, the second has <math>2+3+2</math> dots, and the third has <math>3+4+5+4+3</math> dots. Given the way the hexagons are constructed, it is clear that this pattern continues. Hence, the fourth hexagon has <math>4+5+6+7+6+5+4=\boxed{\textbf{(B) }37}</math> dots.
 
Looking at the rows of each hexagon, we see that the first hexagon has <math>1</math> dot, the second has <math>2+3+2</math> dots, and the third has <math>3+4+5+4+3</math> dots. Given the way the hexagons are constructed, it is clear that this pattern continues. Hence, the fourth hexagon has <math>4+5+6+7+6+5+4=\boxed{\textbf{(B) }37}</math> dots.
  
==Solution 2==
+
== Solution 2 (Pattern of the Bands) ==
The first hexagon has <math>1</math> dot, the second hexagon has <math>1+6</math> dots, the third hexagon <math>1+6+12</math> dots, and so on. The pattern continues since to go from hexagon <math>n</math> to hexagon <math>(n+1),</math> we add a new ring of dots around the outside of the existing ones, with each side of the ring having side length <math>(n+1).</math> Thus the number of dots added is <math>6(n+1)-6 = 6n</math> (we subtract <math>6</math> as each of the corner hexagons in the ring is counted as part of two sides), confirming the pattern. We therefore predict that that the fourth hexagon has <math>1+6+12+18=\boxed{\textbf{(B) }37}</math> dots.
+
The dots in the next hexagon have four bands. From innermost to outermost:
 
 
==Solution 3 (Variant of Solution 2)==
 
 
 
Let the number of dots in the first hexagon be <math>h_0 = 1.</math> By the same argument as in Solution 2, we have <math>h_n=h_{n-1}+6n</math> for <math>n > 0.</math> Using this, we find that <math>h_1=7,h_2=19,</math> and <math>h_3=\boxed{\textbf{(B) }37}.</math>
 
 
 
== Solution 4 (Variant of Solution 2) ==
 
The dots in the next hexagon have four layers. From innermost to outermost:
 
 
<ol style="margin-left: 1.5em;">
 
<ol style="margin-left: 1.5em;">
   <li>The first layer has <math>1</math> dot.</li><p>
+
   <li>The first band has <math>1</math> dot.</li><p>
   <li>The second layer has <math>6</math> dots: <math>1</math> dot at each vertex of the hexagon.</li><p>
+
   <li>The second band has <math>6</math> dots: <math>1</math> dot at each vertex of the hexagon.</li><p>
   <li>The third layer has <math>6+6\cdot1=12</math> dots: <math>1</math> dot at each vertex of the hexagon and <math>1</math> other dot on each edge of the hexagon.</li><p>
+
   <li>The third band has <math>6+6\cdot1=12</math> dots: <math>1</math> dot at each vertex of the hexagon and <math>1</math> other dot on each edge of the hexagon.</li><p>
   <li>The fourth layer has <math>6+6\cdot2=18</math> dots: <math>1</math> dot at each vertex of the hexagon and <math>2</math> other dots on each edge of the hexagon.</li><p>
+
   <li>The fourth band has <math>6+6\cdot2=18</math> dots: <math>1</math> dot at each vertex of the hexagon and <math>2</math> other dots on each edge of the hexagon.</li><p>
 
</ol>
 
</ol>
 
Together, the answer is <math>1+6+12+18=\boxed{\textbf{(B) }37}.</math>
 
Together, the answer is <math>1+6+12+18=\boxed{\textbf{(B) }37}.</math>
Line 55: Line 45:
 
~MRENTHUSIASM
 
~MRENTHUSIASM
  
== Solution 5 (Brute Force) ==
+
==Solution 3 (Pattern of the Bands)==
 +
The first hexagon has <math>1</math> dot, the second hexagon has <math>1+6</math> dots, the third hexagon <math>1+6+12</math> dots, and so on. The pattern continues since to go from hexagon <math>n</math> to hexagon <math>(n+1),</math> we add a new band of dots around the outside of the existing ones, with each side of the band having side length <math>(n+1).</math> Thus the number of dots added is <math>6(n+1)-6 = 6n</math> (we subtract <math>6</math> as each of the corner hexagons in the band is counted as part of two sides.). We therefore predict that that the fourth hexagon has <math>1+6+12+18=\boxed{\textbf{(B) }37}</math> dots.
 +
 
 +
<u><b>Remark</b></u>
 +
 
 +
For positive integers <math>n,</math> let <math>h_n</math> denote the number of dots in the <math>n</math>th hexagon. We have <math>h_1=1</math> and <math>h_{n+1}=h_n+6n.</math>
 +
 
 +
It follows that <math>h_2=7,h_3=19,</math> and <math>h_4=37.</math>
 +
 
 +
== Solution 4 (Brute Force) ==
 
From the full diagram below, the answer is <math>\boxed{\textbf{(B) }37}.</math>
 
From the full diagram below, the answer is <math>\boxed{\textbf{(B) }37}.</math>
 
<asy>
 
<asy>
size(500);
+
// diagram by SirCalcsALot, edited by MRENTHUSIASM
real side1 = 1.5;
+
size(400);
real side2 = 4.0;
+
path p = scale(0.8)*unitcircle;
real side3 = 6.5;
+
pair[] A;
real side4 = 9.0;
 
real pos = 2.5;
 
pair s1 = (-10,-2.19);
 
pair s2 = (15,2.19);
 
pair s3 = (36,4.38);
 
 
pen grey1 = rgb(100/256, 100/256, 100/256);
 
pen grey1 = rgb(100/256, 100/256, 100/256);
 
pen grey2 = rgb(183/256, 183/256, 183/256);
 
pen grey2 = rgb(183/256, 183/256, 183/256);
fill(circle(origin + s1, 1), grey1);
+
for (int i=0; i<7; ++i) { A[i] = rotate(60*i)*(1,0);}
for (int i = 0; i < 6; ++i) {
+
path hex = A[0]--A[1]--A[2]--A[3]--A[4]--A[5]--cycle;
draw(side1*dir(60*i)+s1--side1*dir(60*i-60)+s1,linewidth(1.25));
+
fill(p,grey1);
 +
draw(scale(1.25)*hex,black+linewidth(1.25));
 +
pair S = 6A[0]+2A[1];
 +
fill(shift(S)*p,grey1);
 +
for (int i=0; i<6; ++i) { fill(shift(S+2*A[i])*p,grey2);}
 +
draw(shift(S)*scale(3.25)*hex,black+linewidth(1.25));
 +
pair T = 16A[0]+4A[1];
 +
fill(shift(T)*p,grey1);
 +
for (int i=0; i<6; ++i) {
 +
fill(shift(T+2*A[i])*p,grey2);
 +
fill(shift(T+4*A[i])*p,grey1);
 +
fill(shift(T+2*A[i]+2*A[i+1])*p,grey1);
 
}
 
}
fill(circle(origin, 1), grey1);
+
draw(shift(T)*scale(5.25)*hex,black+linewidth(1.25));
for (int i = 0; i < 6; ++i) {
+
 
fill(circle(pos*dir(60*i),1), grey2);
+
pair R = 30A[0]+6A[1];
draw(side2*dir(60*i)--side2*dir(60*i-60),linewidth(1.25));
+
fill(shift(R)*p,grey1);
}
+
for (int i=0; i<6; ++i) {  
fill(circle(origin+s2, 1), grey1);
+
fill(shift(R+2*A[i])*p,grey2);
for (int i = 0; i < 6; ++i) {
+
fill(shift(R+4*A[i])*p,grey1);
fill(circle(pos*dir(60*i)+s2,1), grey2);
+
fill(shift(R+2*A[i]+2*A[i+1])*p,grey1);
fill(circle(2*pos*dir(60*i)+s2,1), grey1);
+
fill(shift(R+6*A[i+1])*p,grey2);
fill(circle(sqrt(3)*pos*dir(60*i+30)+s2,1), grey1);
+
fill(shift(R+2*A[i]+4*A[i+1])*p,grey2);
draw(side3*dir(60*i)+s2--side3*dir(60*i-60)+s2,linewidth(1.25));
+
fill(shift(R+4*A[i]+2*A[i+1])*p,grey2);
}
 
fill(circle(origin+s3, 1), grey1);
 
for (int i = 0; i < 6; ++i) {
 
fill(circle(pos*dir(60*i)+s3,1), grey2);
 
fill(circle(2*pos*dir(60*i)+s3,1), grey1);
 
fill(circle(sqrt(3)*pos*dir(60*i+30)+s3,1), grey1);
 
fill(circle(3*pos*dir(60*i)+s3,1), grey2);
 
fill(circle(sqrt(7)*pos*dir(60*i+79.1066054)+s3,1), grey2);
 
fill(circle(sqrt(7)*pos*dir(60*i-79.1066054)+s3,1), grey2);
 
draw(side4*dir(60*i)+s3--side4*dir(60*i-60)+s3,linewidth(1.25));
 
 
}
 
}
 +
draw(shift(R)*scale(7.25)*hex,black+linewidth(1.25));
 
</asy>
 
</asy>
 
~MRENTHUSIASM
 
~MRENTHUSIASM
 
== Video Solution ==
 
https://www.youtube.com/watch?v=_IjQnXnVKeU
 
  
 
==Video Solution by WhyMath==
 
==Video Solution by WhyMath==
Line 106: Line 99:
 
~savannahsolver
 
~savannahsolver
  
==Video Solution==
+
==Video Solution by The Learning Royal==
 
https://youtu.be/eSxzI8P9_h8
 
https://youtu.be/eSxzI8P9_h8
 +
 +
~The Learning Royal
  
 
==Video Solution by Interstigation==
 
==Video Solution by Interstigation==
Line 113: Line 108:
  
 
~Interstigation
 
~Interstigation
 +
 +
== Video Solution by North America Math Contest Go Go Go ==
 +
https://www.youtube.com/watch?v=_IjQnXnVKeU
 +
 +
~North America Math Contest Go Go Go
  
 
==See also==  
 
==See also==  
 
{{AMC8 box|year=2020|num-b=3|num-a=5}}
 
{{AMC8 box|year=2020|num-b=3|num-a=5}}
 
{{MAA Notice}}
 
{{MAA Notice}}

Revision as of 02:23, 12 January 2022

Problem

Three hexagons of increasing size are shown below. Suppose the dot pattern continues so that each successive hexagon contains one more band of dots. How many dots are in the next hexagon?

[asy] // diagram by SirCalcsALot, edited by MRENTHUSIASM size(250); path p = scale(0.8)*unitcircle; pair[] A; pen grey1 = rgb(100/256, 100/256, 100/256); pen grey2 = rgb(183/256, 183/256, 183/256); for (int i=0; i<7; ++i) { A[i] = rotate(60*i)*(1,0);} path hex = A[0]--A[1]--A[2]--A[3]--A[4]--A[5]--cycle; fill(p,grey1); draw(scale(1.25)*hex,black+linewidth(1.25)); pair S = 6A[0]+2A[1]; fill(shift(S)*p,grey1); for (int i=0; i<6; ++i) { fill(shift(S+2*A[i])*p,grey2);} draw(shift(S)*scale(3.25)*hex,black+linewidth(1.25)); pair T = 16A[0]+4A[1]; fill(shift(T)*p,grey1); for (int i=0; i<6; ++i) {   fill(shift(T+2*A[i])*p,grey2);  fill(shift(T+4*A[i])*p,grey1);  fill(shift(T+2*A[i]+2*A[i+1])*p,grey1); } draw(shift(T)*scale(5.25)*hex,black+linewidth(1.25)); [/asy]

$\textbf{(A) }35 \qquad \textbf{(B) }37 \qquad \textbf{(C) }39 \qquad \textbf{(D) }43 \qquad \textbf{(E) }49$

Solution 1 (Pattern of the Rows)

Looking at the rows of each hexagon, we see that the first hexagon has $1$ dot, the second has $2+3+2$ dots, and the third has $3+4+5+4+3$ dots. Given the way the hexagons are constructed, it is clear that this pattern continues. Hence, the fourth hexagon has $4+5+6+7+6+5+4=\boxed{\textbf{(B) }37}$ dots.

Solution 2 (Pattern of the Bands)

The dots in the next hexagon have four bands. From innermost to outermost:

  1. The first band has $1$ dot.
  2. The second band has $6$ dots: $1$ dot at each vertex of the hexagon.
  3. The third band has $6+6\cdot1=12$ dots: $1$ dot at each vertex of the hexagon and $1$ other dot on each edge of the hexagon.
  4. The fourth band has $6+6\cdot2=18$ dots: $1$ dot at each vertex of the hexagon and $2$ other dots on each edge of the hexagon.

Together, the answer is $1+6+12+18=\boxed{\textbf{(B) }37}.$

~MRENTHUSIASM

Solution 3 (Pattern of the Bands)

The first hexagon has $1$ dot, the second hexagon has $1+6$ dots, the third hexagon $1+6+12$ dots, and so on. The pattern continues since to go from hexagon $n$ to hexagon $(n+1),$ we add a new band of dots around the outside of the existing ones, with each side of the band having side length $(n+1).$ Thus the number of dots added is $6(n+1)-6 = 6n$ (we subtract $6$ as each of the corner hexagons in the band is counted as part of two sides.). We therefore predict that that the fourth hexagon has $1+6+12+18=\boxed{\textbf{(B) }37}$ dots.

Remark

For positive integers $n,$ let $h_n$ denote the number of dots in the $n$th hexagon. We have $h_1=1$ and $h_{n+1}=h_n+6n.$

It follows that $h_2=7,h_3=19,$ and $h_4=37.$

Solution 4 (Brute Force)

From the full diagram below, the answer is $\boxed{\textbf{(B) }37}.$ [asy] // diagram by SirCalcsALot, edited by MRENTHUSIASM size(400); path p = scale(0.8)*unitcircle; pair[] A; pen grey1 = rgb(100/256, 100/256, 100/256); pen grey2 = rgb(183/256, 183/256, 183/256); for (int i=0; i<7; ++i) { A[i] = rotate(60*i)*(1,0);} path hex = A[0]--A[1]--A[2]--A[3]--A[4]--A[5]--cycle; fill(p,grey1); draw(scale(1.25)*hex,black+linewidth(1.25)); pair S = 6A[0]+2A[1]; fill(shift(S)*p,grey1); for (int i=0; i<6; ++i) { fill(shift(S+2*A[i])*p,grey2);} draw(shift(S)*scale(3.25)*hex,black+linewidth(1.25)); pair T = 16A[0]+4A[1]; fill(shift(T)*p,grey1); for (int i=0; i<6; ++i) {   fill(shift(T+2*A[i])*p,grey2);  fill(shift(T+4*A[i])*p,grey1);  fill(shift(T+2*A[i]+2*A[i+1])*p,grey1); } draw(shift(T)*scale(5.25)*hex,black+linewidth(1.25));  pair R = 30A[0]+6A[1]; fill(shift(R)*p,grey1); for (int i=0; i<6; ++i) {   fill(shift(R+2*A[i])*p,grey2);  fill(shift(R+4*A[i])*p,grey1);  fill(shift(R+2*A[i]+2*A[i+1])*p,grey1);  fill(shift(R+6*A[i+1])*p,grey2);  fill(shift(R+2*A[i]+4*A[i+1])*p,grey2);  fill(shift(R+4*A[i]+2*A[i+1])*p,grey2); } draw(shift(R)*scale(7.25)*hex,black+linewidth(1.25)); [/asy] ~MRENTHUSIASM

Video Solution by WhyMath

https://youtu.be/szWgrOPNw8c

~savannahsolver

Video Solution by The Learning Royal

https://youtu.be/eSxzI8P9_h8

~The Learning Royal

Video Solution by Interstigation

https://youtu.be/YnwkBZTv5Fw?t=123

~Interstigation

Video Solution by North America Math Contest Go Go Go

https://www.youtube.com/watch?v=_IjQnXnVKeU

~North America Math Contest Go Go Go

See also

2020 AMC 8 (ProblemsAnswer KeyResources)
Preceded by
Problem 3
Followed by
Problem 5
1 2 3 4 5 6 7 8 9 10 11 12 13 14 15 16 17 18 19 20 21 22 23 24 25
All AJHSME/AMC 8 Problems and Solutions

The problems on this page are copyrighted by the Mathematical Association of America's American Mathematics Competitions. AMC logo.png